Difference between revisions of "2025 SSMO Speed Round Problems/Problem 2"
(→Solution) |
m (→Solution) |
||
Line 9: | Line 9: | ||
A=(0,0); | A=(0,0); | ||
B=(50,0); | B=(50,0); | ||
− | + | N=(35,0); | |
− | + | M=(15,0); | |
draw(A--B); | draw(A--B); | ||
Line 25: | Line 25: | ||
</asy> | </asy> | ||
− | Note that <math>AN+BM = AB | + | Note that <math>AN+BM = AB + MN = 70</math>. By AM-GM, the value of <math>AN\cdot BM</math> is at a maximum when <math>AN = BM = 35</math>. Thus, <math>AM = AN-MN = \boxed{15}</math>. |
~Sedro | ~Sedro |
Latest revision as of 15:02, 9 September 2025
Problem
Let and
be points such that
. Points
and
lie on
such that
lies between points
and
and
lies between points
and
. Given that
and
is maximized, find the length of
.
Solution
Note that . By AM-GM, the value of
is at a maximum when
. Thus,
.
~Sedro